Using differentials to estimate the amount of tin

Click For Summary
To estimate the amount of tin in a closed tin can using differentials, the maximum volume is calculated as π(4)(12), while the enclosed volume is derived from the reduced radius due to thickness. The differential volume formula is dv = 2πrh dr + πr² dh, where dr is determined as 0.4 cm and dh is suggested to be 0.08 cm to account for both the top and bottom lids. This approach yields an estimated volume of tin around 16.08 cm³, which aligns closely with the book's answer. The discussion emphasizes the importance of correctly applying the differential method to achieve accurate results.
tnutty
Messages
324
Reaction score
1

Homework Statement



Use differentials to estimate the amount of tin in a closed tin can with diameter 8cm, and height 12cm, if the tin is 0.04 cm thick.

The way I did this was to subtract the Max volume by the Enclosed volume.

Max Volume is pi(4)(12)
Enclosed Volume is pi ( 8 - 2(0.04) )/2 *12

I got about 12cm^3How do I do this with differentials ?V = pi*r^2*h

dv = 2pi*rh * dr + pi*r^2 dh

Then what?

I think dr = ( 8 - (8 - 2*0.4) ) / 2 = 0.4
Then what is dh ?

Do I have to compare it to the Total volume somehow?
 
Last edited:
Physics news on Phys.org
tnutty said:

Homework Statement



Use differentials to estimate the amount of tin in a closed tin can with diameter 8cm, and height 12cm, if the tin is 0.04 cm thick.

The way I did this was to subtract the Max volume by the Enclosed volume.

Max Volume is pi(4)(12)
Enclosed Volume is pi ( 8 - 2(0.04) )/2 *12

I got about 12cm^3


How do I do this with differentials ?


V = pi*r^2*h

dv = 2pi*r * dr + pi*r^2 dh

Then what?

I think dr = ( 8 - (8 - 2*0.4) ) / 2 = 0.4
Then what is dh ?
It seems to me that dh = 2* 0.4, since you have two lids (top and bottom) of thickness 0.4 cm each.
tnutty said:
Do I have to compare it to the Total volume somehow?
No, your expression for dV should do the trick.
 
No, the answer in the book says that its 16cm^3.

The first method I did, subtracting the max volume by the volume with radius 1/2( 8 - 2*0.04) was close because the radius will be 3.6, from the answer(16cm^3) the radius has to be close to 3.5.

I am not sure how to work with differentials with this problem though.
 
Using dh = .08, I get
dV \approx~4\pi(24(.04) + 4(.08))~\approx~16.08495 cm^3
If I use dh = .04, I get an answer that is about 14.07 cm^3

My answer using dh = .08 is a lot closer to your book's answer.
 
Mark44 said:
Using dh = .08, I get
dV \approx~4\pi(24(.04) + 4(.08))~\approx~16.08495 cm^3
If I use dh = .04, I get an answer that is about 14.07 cm^3

My answer using dh = .08 is a lot closer to your book's answer.


Sorry for my miscalculation, thanks a million, Mark.
 
Question: A clock's minute hand has length 4 and its hour hand has length 3. What is the distance between the tips at the moment when it is increasing most rapidly?(Putnam Exam Question) Answer: Making assumption that both the hands moves at constant angular velocities, the answer is ## \sqrt{7} .## But don't you think this assumption is somewhat doubtful and wrong?

Similar threads

  • · Replies 2 ·
Replies
2
Views
16K
Replies
1
Views
2K
  • · Replies 6 ·
Replies
6
Views
7K
  • · Replies 3 ·
Replies
3
Views
3K
  • · Replies 9 ·
Replies
9
Views
2K
  • · Replies 2 ·
Replies
2
Views
2K
  • · Replies 3 ·
Replies
3
Views
5K
  • · Replies 7 ·
Replies
7
Views
2K
  • · Replies 2 ·
Replies
2
Views
4K
  • · Replies 2 ·
Replies
2
Views
3K